Đến nội dung

sinh vien nội dung

Có 261 mục bởi sinh vien (Tìm giới hạn từ 29-04-2020)



Sắp theo                Sắp xếp  

#559232 Các bài toán về hàm hình học

Đã gửi bởi sinh vien on 13-05-2015 - 21:07 trong Hình học

Bài toán (Putnam 2009). Gọi $\mathbb{P}$ là tập hợp các điểm trên mặt phẳng , $f:\mathbb{P}\rightarrow \mathbb{R}$ là một hàm số nhận giá trị thực thỏa mãn $f(A)+f(B)+f(C)+f(D)=0$, với mọi hình vuông ABCD .

    Chứng minh rằng f(P)=0 với mọi P

Lời giải. Gọi P là một điểm bất kỳ thuộc $\mathbb{P}$. Gọi ABCD là hình vuông nhận P làm tâm . Gọi E,F,G,H lần lượt là trung điểm của các cạnh AB ,BC,CD,DA. 

Từ điều kiện đề bài ta có 

   0=f(A)+f(B)+f(C)+f(D)

   0=f(E)+f(F)+f(G)+f(H)

   0=f(A)+f(E)+f(P)+f(H)

   0=f(B)+f(F)+f(P)+f(E)

   0=f(C)+f(G)+f(P)+f(F)

   0=f(D)+f(H)+f(P)+f(G)

 Cộng bồn đẳng thức cuối kết hợp với hai đẳng thức đầu ta có ngay kết quả




#560140 Bài toán tô màu các số

Đã gửi bởi sinh vien on 18-05-2015 - 12:24 trong Thảo luận về các kì thi, các kì kiểm tra Toán sinh viên

Bài toán. Ta tô màu các số thuộc tập hợp {1,2,...,n} bằng một trong 6 màu khác nhau.

 Đặt

       $S=\left \{ (x,y,z)\in \left \{ 1,2...n \right \}^{3}:x+y+z\equiv 0\right \}$ và x,y,z có cùng màu

        $D=\left \{ (x,y,z)\in \left \{ 1,2,...n \right \}^{3} :x+y+z\equiv 0\right \}$ và x,y,z có màu đôi một khác nhau

  Chứng minh rằng : $\left | D \right |\leq 2\left | S \right |+\frac{n^{2}}{2}$

 Nguồn : Từ cuộc thi Annual Vojtech Jarnik




#719593 bài toán năm vật thể đều

Đã gửi bởi sinh vien on 19-01-2019 - 22:20 trong Tài liệu, chuyên đề Toán cao cấp

File gửi kèm  Bài toán năm vật thể đều.pdf   342.4K   77 Số lần tải




#559330 Một minh họa thú vị

Đã gửi bởi sinh vien on 14-05-2015 - 17:16 trong Giải tích

Bài toán.(Putnam 1958) Chứng minh rằng phương trình tích phân sau nếu  có nghiệm  thuộc lớp hàm liên tục trên hình vuông $[0,1]\times [0,1]$.

                                $f(x,y)=1+\int_{0}^{x}\int_{0}^{y}f(u,v)dudv$

thì nghiệm đó là duy nhất.

Lời giải . Gỉa sử $f_{1}(x,y),f_{2}(x,y)$ là hai nghiệm bất kỳ của phương trình tích phân đã cho. Đặt $g(x,y)=f_{1}(x,y)-f_{2}(x,y)$. Ta thấy

$g(x,y)=\left ( 1+\int_{0}^{x}\int_{0}^{y}f_{1}(u,v)dudv \right )-\left ( 1+\int_{0}^{x}\int_{0}^{y}f_{2}(u,v)dudv \right )$

          $=\int_{0}^{x}\int_{0}^{y}\left ( f_{1}(u,v)-f_{2}(u,v) \right )dudv=\int_{0}^{x}\int_{0}^{y}g(x,y)dxdy$

Dễ dàng nhận thấy g(x,y) cũng là một hàm liên tục trên $[0,1]\times [0,1]$ nên g(x,y) bị chặn

hay nói cách khác tồn tại M sao cho $\left | g(x,y) \right |\leq M$ với mọi $(x,y)\in [0,1]\times [0,1]$

 Ta sẽ chứng minh quy nạp kết quả then chốt sau : $\left | g(x,y) \right |\leq M\frac{x^{n}}{n!}\frac{y^{n}}{n!}$ với mọi số nguyên dương n

 Với n=1 , kết quả hiển nhiên.

Gỉa sử kết quả đúng với n=k . Ta cần chứng minh nó cũng đúng cho n=k+1. Ta thấy

$\left | g(x,y) \right |\leqslant \left | \int_{0}^{x}\int_{0}^{y}g(u,v)dudv \right |\leq \int_{0}^{x}\int_{0}^{y}M\frac{u^{k}}{k!}\frac{v^{k}}{k!}dudv=M\frac{x^{k+1}}{(k+1)!}\frac{y^{k+1}}{(k+1)!}$

 Vậy kết quả cũng đúng với n=k+1

  Từ kết quả trên nếu ta cố định bộ số (x,y) , qua giới hạn khi $n\rightarrow \infty$ , ta suy ra $\left | g(x,y) \right |\leq 0\Rightarrow g(x,y)=0$

    Nhận xét này chứng tỏ rằng phương trình tích phân đã cho không thể có hai nghiệm phân biệt .




#557834 CMR tồn tại hằng số B dương thỏa $\sum_{i,j=1}^{n...

Đã gửi bởi sinh vien on 04-05-2015 - 18:56 trong Giải tích

Bài toán. (PUTNAM 2011) Cho $a_{1},a_{2},...a_{n}$ là các số thực thỏa mãn 

          $\int_{-\infty }^{\infty }\left ( \sum_{i=1}^{n}\frac{1}{1+\left ( x-a_{i} \right )^{2}}\right )^{2}dx\leqslant An$ với mọi n , trong đó A là một hằng số dương .

 Chứng minh rằng tồn tại hằng số B dương thỏa $\sum_{i,j=1}^{n}\left ( 1+\left ( a_{i}-a_{j} \right )^{2} \right )\geq Bn^{3}$.

Lời giải. 

 Với $a\neq 0$, $\int_{-\infty }^{\infty }\frac{du}{\left ( 1+\left ( u+a \right )^{2} \right )\left ( 1+\left ( u-a \right )^{2} \right )}=\frac{1}{4a(1+a^{2})}\int_{-\infty }^{\infty }\left [ \frac{u+a}{1+\left ( u+a\right )^{2}}-\frac{u-a}{1+\left ( u-a \right )^{2}} \right ]du$$=\frac{1}{4a\left ( 1+a^{2} \right )}\left [ \frac{1}{2}ln\frac{1+\left ( u+a \right )^{2}}{1+(u-a)^{2}} +aarctan(u+a)+aarctan(u-a)\right ]_{-\infty }^{\infty }$$=\frac{\pi }{2(1+a^{2})}$.

   Sử dụng phép đổi biến: $x=u+\frac{a+b}{2}$ ta được 

$\int_{-\infty }^{\infty }\frac{dx}{(1+(x-a)^{2})(1+(x-b)^{2})}=\frac{2\pi }{4+(a-b)^{2}}$, trong đó $a\neq b$

Trong trường hợp a=b 

  $\int_{-\infty }^{\infty }\frac{dx}{(1+(x-a)^{2})^{2}}=\int_{-\frac{\pi }{2}}^{\frac{\pi }{2}}\frac{\frac{1}{cos^{2}\theta }d\theta }{\frac{1}{cos^{4}\theta }}=\int_{-\frac{\pi }{2}}^{\frac{\pi }{2}}cos^{2}\theta d\theta =\frac{\pi }{2}$

 nên côn thức trên cũng đúng với mọi cặp số a,b.

 Ta có

$An\geq \int_{-\infty }^{\infty }\left [ \sum_{i=1}^{n}\frac{1}{1+(x-a_{i})^{2}}\right ]^{2}dx=\sum_{i,j=1}^{n}\int_{-\infty }^{\infty }\frac{dx}{(1+(x-a_{i})^{2})(1+(x-a_{j})^{2})}$$=\sum_{i,j=1}^{n}\frac{2\pi }{4+(a_{i}-a_{j})^{2}}\geq \frac{\pi }{2}\sum_{i,j=1}^{n}\frac{1}{1+\left ( a_{i}-a_{j} \right )^{2}}$

 Sử dụng bất đẳng thức quen thuộc Cauchy- Schwarz 

$n^{4}\leq \left [ \sum_{i,j=1}^{n}\frac{1}{1+(a_{i}-a_{j})^{2}} \right ]\sum_{i,j=1}^{n}(1+(a_{i}-a_{j})^{2})\leq \frac{2An}{\pi }\sum_{i,j=1}^{n}(1+(a_{i}-a_{j})^{2})$

  Chọn $B=\frac{\pi }{2A}$ . thì điều kiện bài toán được thỏa mãn




#688167 Đường đi ngẫu nhiên trên đồ thị

Đã gửi bởi sinh vien on 20-07-2017 - 17:28 trong Tài liệu, chuyên đề Toán cao cấp

File gửi kèm  Đường đi ngẫu nhiên trên đồ thị.pdf   292.76K   100 Số lần tải




#560220 Sử dụng tích phân chứng minh bất đẳng thức

Đã gửi bởi sinh vien on 18-05-2015 - 20:18 trong Giải tích

Bài toán(Hilbert).Chứng minh :

  Với mọi số thực $a_{1},a_{2},...,a_{n}$ ta luôn có

                   $\sum_{i=1}^{n}\sum_{j=1}^{n}\frac{a_{i}a_{j}}{i+j}\leq \pi \sum_{i=1}^{n}a_{i}^{2}$

Bài toán ( Frilz Carlson) Với mọi số thực $a_{1},a_{2},...,a_{n}$ ta luôn có

                          $\pi ^{2}(a_{1}^{2}+a_{2}^{2}+...+a_{n}^{2})(a_{1}^{2}+4a_{2}^{2}+...+n^{2}a_{n}^{2})\leq (a_{1}+a_{2}+...+a_{n})^{4}$

Bài toán.  Chứng minh rằng với a,b,c,x,y,z,t là các số thực dương sao cho $1\leq x,y,z\leq 4$ ta luôn có

       $\frac{x}{(2a)^{t}}+\frac{y}{(2b)^{t}}+\frac{z}{(2c)^{t}}\geq \frac{y+z-x}{(b+c)^{t}}+\frac{z+x-y}{(c+a)^{t}}+\frac{x+y-z}{(a+b)^{t}}$

   Nguồn các bài toán : 2 bài đầu mình trích ra từ cuốn '' Problem from Book '' của tác giả Titu Andresscu.

Bài toán còn lại là từ một cuộc thi tại cuộc thi Annual Vojtech Jarnik




#560962 Bài toán tô màu - hệ phương trình

Đã gửi bởi sinh vien on 22-05-2015 - 19:23 trong Thảo luận về các kì thi, các kì kiểm tra Toán sinh viên

Bài toán (AVJ-2009) Cho k và n là hai số nguyên dương thỏa $k\leq n-1$ . Đặt $S=\left \{ 1,2,...,n \right \}$ và $A_{1},A_{2},..A_{k}$ là các tập con khác rỗng của S. Chứng minh rằng ta có thể tô màu một số phần tử của S bằng một trong hai màu xanh hoặc đỏ ( có thể có những phần tử không được tô màu) sao cho

 i) Một phần tử bất kỳ của S hoặc không được tô màu hoặc được tô màu đỏ hoặc được tô màu xanh

 ii) Có ít nhất một phần tử của S được tô màu

 iii) Mọi tập $A_{i} (i=1,2...k)$ hoặc chứa toàn các số không được tô màu hoặc chứa ít nhất một cặp số mà mỗi số được tô bởi hai màu khác nhau




#562123 Tính số chiều của các không gian các vector con

Đã gửi bởi sinh vien on 28-05-2015 - 16:45 trong Đại số tuyến tính, Hình học giải tích

Bài toán.(ĐH - Toronto 2015 ) Tính số chiều của không gian vector con sinh bởi $\left \{ (\sigma (1);\sigma (2);...;\sigma (n)) \right \}$. trong đó $\sigma$  là một song ánh từ $\left \{ 1,2...n \right \}\rightarrow \left \{ 1,2,...,n \right \}$.

   Đáp số : n




#557802 Sự kết hợp giữa tích phân và số phức

Đã gửi bởi sinh vien on 04-05-2015 - 08:08 trong Giải tích

Bài toán. Chứng minh rằng số điểm nguyên trong hình lập phương đóng $-n\leq x,y,z\leq n$ thỏa mãn điều kiện $-s\leq x+y+z\leq s$ bằng $\frac{1}{2\pi }\int_{-\pi }^{\pi }\left ( \frac{sin\frac{2n+1}{2}t}{sin\frac{t}{2}} \right )^{3}\frac{sin\frac{2s+1}{2}t}{sin\frac{t}{2}}dt$ trong đó s, n nguyên

Lời giải. Xét hàm sinh của bài toán

     $G(x)=\left ( \frac{1}{x^{n}}+\frac{1}{x^{n-1}}+...+\frac{1}{x}+1+x+...+x^{n-1}+x^{n} \right )^{3}$

  $=...+a_{-k}x^{-k}+a_{-k+1}x^{-k+1}+...+a_{-1}x^{-1}+a_{0}+a_{1}x+...+a_{k-1}x^{k-1}+a_{k}x^{k}+...$

 Dễ thấy khi đó số các điểm nguyên của hình lập phương đóng thỏa mãn điều kiện $x+y+z=m$ trong đó m nguyên và $-3n\leq m\leq 3n$ chính là hệ số $a_{m}$ trong khai triển trên.

 Do đó 

  Số cần tìm = $a_{-s}+a_{-s+1}+...a_{1}+a_{0}+a_{1}+...+a_{s-1}+a_{s}$

 Để tính tổng này ta chú ý đến một kết quả quen thuộc trong giải tích phức:

                         $\int_{-\pi }^{\pi }e^{ikx}dx=2\pi$ nếu k=0 và bằng 0 trong các trường hợp còn lại.

nên nói chung ta thấy:

$\frac{1}{2\pi }\int_{-\pi }^{\pi }G(e^{it})e^{-kxt}dt=a_{k}$, trong đó$-3n\leq k\leq 3n$

 Để thuận tiện trong trình bày ta sẽ đặt $\zeta =e^{it}$ nên ta được :

         $\frac{1}{2\pi }\int_{-\pi }^{\pi }G(\zeta)\zeta ^{-k}dt=a_{k}$

 Ta có

  $\sum_{i=-s}^{s}a_{i}=\frac{1}{2\pi }\int_{-\pi }^{\pi }G(\zeta)(\sum_{i=-s}^{s}\zeta ^{i})dt$

  Không quá khó khăn để chứng minh đẳng thức sau:

                 $\sum_{i=-m}^{m}\zeta ^{i}=\frac{\zeta ^{-\frac{2m+1}{2}}-\zeta ^{-\frac{2m+1}{2}}}{\zeta ^{-\frac{1}{2}}-\zeta ^{\frac{1}{2}}}=\frac{sin\frac{2m+1}{2}t}{sin\frac{t}{2}}$   ( chú ý ta có: $\zeta =e^{it}$

nên $\sum_{i=-s}^{s}a_{i}=\frac{1}{2\pi }\int_{-\pi }^{\pi }\left ( \frac{sin\frac{2n+1}{2}t}{sin\frac{t}{2}} \right )^{3}\frac{sin\frac{2s+1}{2}t}{sin\frac{t}{2}}dt$ (đpcm)




#559348 Tính tổng của chuỗi bằng phương trình vi phân

Đã gửi bởi sinh vien on 14-05-2015 - 18:18 trong Giải tích

Bài toán (Putnam 1950) Chứng minh rằng:

         $x+\frac{x^{3}}{1.3}+\frac{x^{5}}{1.3.5}+\frac{x^{7}}{1.3.5.7}+...+\frac{x^{2k+1}}{1.3.5...(2k+1)}+...=e^{\frac{x^{2}}{2}}\int_{0}^{x}e^{-\frac{t^{2}}{2}}dt$




#557831 Một sự kết hợp giữa đại số tuyến tính và tổ hợp

Đã gửi bởi sinh vien on 04-05-2015 - 18:29 trong Đại số tuyến tính, Hình học giải tích

Bài toán.  Cho $k\leq \frac{n}{2}$ và $F$ là một họ các ma trận con của một ma trận $n\times n$  sao cho hai ma trận con bất kì đều giao nhau ( có những phần tử chung ) thì $\left | F \right |\leq \left ( C_{n-1}^{k-1} \right )^{2}$, trong đó $\left | A \right |$ kí hiệu số phần tử của tập A.

Lời giải.

Với mọi ma trận con M thuộc họ F, đặt $R_{M};C_{M}$ là các bộ k- số  chỉ thứ tự của các hàng và các cột . Dể dàng nhận thấy $R_{M};C_{M}$ xác định duy nhất ma trận M. Theo giả thiết của bài toán

      $R_{M_{1}}\bigcap R_{M_{2}}\neq \varnothing ;C_{M_{1}}\bigcap C_{M_{2}}\neq \varnothing$ trong đó $M_{1};M_{2}$ là hai phần tử bất kì thuộc họ F.

 Khi đó nếu đặt   $R=\left \{ R_{M};M\in F \right \}$;$C=\left \{ C_{M};M\in F \right \}$ là hai  họ con chứa các bộ k -số lấy từ n số sao cho hai bộ k- số bất kỳ đều có phần tử chung .

   Theo định lý Erdos-Ko-Rado ( xem tuyển tập các chuyên đề tổ hợp ) ta thấy

                 $\left | R \right |\leq C_{n-1}^{k-1};\left | C \right |\leq C_{n-1}^{k-1}\textrm{}$

Theo nhận xét trên ta suy ra$\left | F \right |\leq \left ( C_{n-1}^{k-1}\right )^{2}$ (đpcm )




#555596 Tính tích phân dựa vào phương trình vi phân

Đã gửi bởi sinh vien on 22-04-2015 - 09:20 trong Giải tích

Bài toán .Tính tích phân :

                       $\int_{0}^{\infty }e^{-\frac{t^{2}}{2}}cos\frac{x^{2}}{2t^{2}}dt$

Lời giải:

Ta thấy hàm 

  $y(x)=\int_{0}^{\infty }e^{-\frac{t^{2}}{2}}cos\frac{x^{2}}{2t^{2}}dt$ thỏa mãn phương trình vi phân

$y^{(iv)}+y=0$. Thật vậy:

$y^{'}=\int_{0}^{\infty }e^{-\frac{t^{2}}{2}}sin\frac{x^{2}}{2t^{2}}\frac{-x}{t^{2}}dt=-\int_{0}^{\infty }e^{-\frac{x^{2}}{2u^{2}}}sin\frac{u^{2}}{2}du$

và $y^{''}=-\int_{0}^{\infty }e^{-\frac{x^{2}}{2u^{2}}}sin\frac{u^{2}}{2}\frac{-x}{u^{2}}du=\int_{0}^{\infty }e^{-\frac{t^{2}}{2}}sin\frac{x^{2}}{2t^{2}}dt$

 tiếp tục quá trình tính toán này ta sẽ thu được

   $y^{(iv)}=-\int_{0}^{\infty }e^{-\frac{t^{2}}{2}}cos\frac{x^{2}}{2t^{2}}dt$

các kết quả trên chứng minh nhận định của ta.

   Dựa vào lý thuyết tổng quát về phương trình vi phân tuyến tính hệ số hằng ta thấy, nghiệm tổng quát của phương trình vi phân đã dẫn là :

 $y(x)=e^{\frac{x}{\sqrt{2}}}(C_{1}cos\frac{x}{\sqrt{2}}+C_{2}sin\frac{x}{\sqrt{2}})+e^{-\frac{x}{\sqrt{2}}}(C_{3}cos\frac{x}{\sqrt{2}}+C_{4}sin\frac{x}{\sqrt{2}})$

  Để tính tích phân ở đầu bài ta đi tìm một nghiệm riêng của phương trình vi phân thỏa mãn $y(0)=\int_{0}^{\infty }e^{-\frac{t^{2}}{2}}dt=\sqrt{\frac{\pi }{2}}$ ( tích phân dạng Gauss) , $y^{'}(0)=-\int_{0}^{\infty }sin\frac{u^{2}}{2}du=-\frac{\sqrt{\pi }}{2}$ ( tích phân  dạng  Fresenel ),$y^{''}(0)=0$ và $y^{'''}(0)=\int_{0}^{\infty }cos\frac{u^{2}}{2}du=\frac{\sqrt{\pi }}{2}.$. Dựa vào các điều kiện ban đầu này ta dễ dàng tính được: $C_{1}=C_{2}=C_{3}=0$ và $C_{4}=\sqrt{\frac{\pi }{2}}$

 Do vậy ta thấy : 

                                $\int_{0}^{\infty }e^{-\frac{t^{2}}{2}}cos\frac{x^{2}}{2t^{2}}dt=\sqrt{\frac{\pi }{2}}e^{-\frac{x}{\sqrt{2}}}cos\frac{x}{\sqrt{2}}$

    Ví dụ trên tương đối đơn giản và là một hình dung cụ thể cho thủ thuật tính tích phân thông qua phương trình vi phân




#690567 Đồ thị Expand

Đã gửi bởi sinh vien on 15-08-2017 - 09:34 trong Tài liệu, chuyên đề Toán cao cấp

File gửi kèm  Đồ thị Expand.pdf   255.19K   81 Số lần tải




#560283 Sử dụng định lý xấp xỉ của Wierstrass để tính giới hạn

Đã gửi bởi sinh vien on 19-05-2015 - 09:22 trong Giải tích

Bài toán. Cho $f:[0,1]\times [0,1]\rightarrow \mathbb{R}$ là một hàm số liên tục

Tính $lim_{n\rightarrow \infty }\left ( \frac{(2n+1)!}{ (n!)^{2}} \right )^{2}\int_{0}^{1}\int_{0}^{1}(xy(1-x) (1-y))^{n}f(x,y)dxdy$

  Nguồn gốc : Từ cuộc thi Annual Vojtech Jarnik




#719598 Công thức Cramer - Định thức

Đã gửi bởi sinh vien on 19-01-2019 - 22:36 trong Tài liệu, chuyên đề Toán cao cấp

File gửi kèm  Công thức Cramer – Định thức.pdf   417.54K   197 Số lần tải




#690610 Đánh giá số cạnh của một đồ thị phẳng

Đã gửi bởi sinh vien on 15-08-2017 - 20:53 trong Tài liệu, chuyên đề Toán cao cấp

File gửi kèm  Đánh giá số cạnh của một đồ thị phẳng.pdf   159.06K   120 Số lần tải




#558170 Hệ phương trình vi phân loại khó

Đã gửi bởi sinh vien on 07-05-2015 - 11:55 trong Giải tích

Bài toán ( Putnam 2009) . Gỉa sử $f,g,h$ là các hàm khả vi trên một khoảng mở chứa điểm 0 Và thỏa mãn :

$f^{'}=2f^{2}gh+\frac{1}{gh};g^{'}=fg^{2}h+\frac{4}{fh};h^{'}=3fgh^{2}+\frac{1}{fg}$ 

 và $f(0)=1;g(0)=1;h(0)=1$.

 Tìm một biểu thức cho hàm f(x) , giả thiết f(x) trong triệt tiêu trên khoảng mở chứa 0.

Lời giải. Nhân đẳng thức thứ nhất cho gh , thứ 2 cho fh, và đẳng thức cuối cho fg ta được

    $f^{'}gh=2\left ( fgh \right )^{2}+1;fg^{'}h=(fgh)^{2}+4;fgh^{'}=3(fgh)^{2}+1$.

 Cộng các đẳng thức này ta được

    $f^{'}gh+fg^{'}h+fgh^{'}=6(fgh)^{2}+6\Rightarrow (fgh)^{'}=6(fgh)^{2}+6$.

 Bằng cách đặt k(x)=f(x)g(x)h(x); ta được $k^{'}=6k^{2}+6\Rightarrow \frac{\mathrm{d} k}{\mathrm{d} x}=6k^{2}+6\Rightarrow \frac{dk}{6k^{2}+6}=dx$

 Kết hợp với điều kiện k(0)=1 ta thấy: $k(x)=tan\left ( 6x+\frac{\pi }{4} \right ).$

   Lại thấy:$\frac{f^{'}}{f}=2tan\left ( 6x+\frac{\pi }{4} \right )+cot\left ( 6x+\frac{\pi }{4} \right )$

$\Rightarrow lnf(x)=\frac{-2lncos\left ( 6x+\frac{\pi }{4} \right )+lnsin\left ( 6x+\frac{\pi }{4} \right )}{6}$

$\Rightarrow f(x)=e^{c}\left ( \frac{sin\left ( 6x+\frac{\pi }{4} \right )}{cos^{2}\left ( 6x+\frac{\pi }{4} \right )} \right )^{\frac{1}{6}}$. Từ giả thiết f(0)=1 ta tính được

   $f(x)=\frac{1}{2^{12}}\left ( \frac{sin\left ( 6x+\frac{\pi }{4} \right )}{cos^{2}\left ( 6x+\frac{\pi }{4} \right )} \right )^{\frac{1}{6}}$.

  Lưu ý là ta cũng có thể tính được g(x), h(x) bằng các tương tự.




#688645 Các bài toán về họ tập có tính chất giao

Đã gửi bởi sinh vien on 25-07-2017 - 18:36 trong Tài liệu, chuyên đề Toán cao cấp

File gửi kèm  Các bài toán về họ có tính chất giao.pdf   282.26K   254 Số lần tải




#560733 Sử dụng '' chiều '' để giải các bài toán đại số tuyến tính

Đã gửi bởi sinh vien on 21-05-2015 - 17:35 trong Đại số tuyến tính, Hình học giải tích

Bài toán ( AVJ-?) Cho M là ma trận

                     $M=\begin{pmatrix} -1 & 3 & 0 & ... &0 \\ 3 & 2 & -1 & ... &... \\ 0 & -1 & 2 &... & ...\\ ...& ... & ... &... &... \\ 0 & ... & ... &-1 &2 \end{pmatrix}$

 Chứng minh rằng M có đúng 9 giá trị riêng dương ( tính luôn cả trường hợp bội )




#560854 Một số bài toán hình học giải tích đẹp

Đã gửi bởi sinh vien on 22-05-2015 - 09:39 trong Đại số tuyến tính, Hình học giải tích

Bài toán ( Sydney -1999) Chứng minh rằng:

Tích diện tích của hai tam giác cùng nằm trong một mặt phẳng có các đỉnh $A_{1},A_{2},A_{3}.$ và$B_{1},B_{2},B_{3}.$ bằng

                            $\pm\frac{1}{16}\begin{vmatrix} d_{1,1}^{2} &d_{1,2}^{2} &d_{1,3}^{2} & 1\\d_{2,1}^{2} & d_{2,2}^{2} & d_{2,3}^{2} &1 \\d_{3,1}^{2} & d_{3,2}^{2} &d_{3,3}^{2} &1 \\1 & 1 &1 &0 \end{vmatrix}$

  trong đó $d_{i,j}$ là khoảng cách từ $A_{i}$ tới $B_{j}$




#688401 Bài toán girth cho đồ thị

Đã gửi bởi sinh vien on 23-07-2017 - 12:11 trong Tài liệu, chuyên đề Toán cao cấp

File gửi kèm  Bài toán girth cho đồ thị.pdf   256.08K   89 Số lần tải




#562130 Hàm xác định trên một ma trận

Đã gửi bởi sinh vien on 28-05-2015 - 17:35 trong Đại số tuyến tính, Hình học giải tích

Bài toán ( Seemous  2008 ) Cho $M_{n}(\mathbb{R})$ là tập hợp các ma trận vuông cấp n trên trường số thực. Tìm tất cả các hàm $f:M_{n}(\mathbb{R})\rightarrow \left \{ 0,1,2...,n \right \}$ thỏa mãn 

   $f(XY)\leqslant min\left \{ f(X),f(Y) \right \},\forall X,Y\in M_{n}(\mathbb{R})$

Đáp số : $f(A)=rank(A)$




#719592 bài toán 15 nữ sinh của Kirkman

Đã gửi bởi sinh vien on 19-01-2019 - 22:17 trong Tài liệu, chuyên đề Toán cao cấp

File gửi kèm  Bài toán 15 nữ sinh của Kirkman.pdf   261.18K   123 Số lần tải




#559720 Định thức và lý thuyết đồng dư

Đã gửi bởi sinh vien on 16-05-2015 - 09:48 trong Đại số tuyến tính, Hình học giải tích

Bài toán (Putnam 2002) Cho p là một số nguyên tố,x,y,z là các số thực bất kỳ

 Chứng minh rằng$det\begin{bmatrix} x & y & z\\x^{p} &y^{p} &z^{p} \\ x^{p^{2}} & y^{p^{2}} &z^{p^{2}} \end{bmatrix}\equiv \prod_{i} (a_{i}x+b_{i}y+c_{i}z)(modp)$

 trong đó $a_{i},b_{i},c_{i}$ là các số nguyên nào đó.

  ( Lưu ý rằng ở đây ta hiểu hai đa thức đồng dư nhau theo modulo p nếu các hệ số cùng bậc của chúng đồng dư nhau theo modulo p )